Gönderen Konu: Diyafont Denklemler Çalışma Soruları ($138$ Tane)  (Okunma sayısı 18171 defa)

Çevrimdışı AtakanCİCEK

  • G.O Demirbaş Üye
  • ******
  • İleti: 264
  • Karma: +4/-0
  • Manisa
Ynt: Diyafont Denklemler Çalışma Soruları (70 Tane)
« Yanıtla #15 : Temmuz 18, 2019, 02:42:28 ös »
$21)$

$x-y^4=4$

$x=y^4+4$

$x=y^4+4y^2+4-4y^2=(y^2+2)^2-(2y)^2$ yani $(y^2-2y+2).(y^2+2y+2)=x$ olur. $y^2+2y+2$ için $\bigtriangleup<0$ ve başkatsayı pozitif olduğundan dolayı daima pozitiftir. dolayısıyla $y^2-2y+2$ de pozitiftir. Bu çarpanlardan herhangi biri $1$ olmazsa $x$ asal sayı olamaz.

$y^2-2y+2=1$ olursa $y=1$ $x=5$

$y^2+2y+2=1$ olursa $y=-1$ $x=5$ çözümleri elde edilir. $(5,1),(5,-1)$ çözümleri elde edilir.
Bir matematikçi sanmaz fakat bilir, inandırmaya çalışmaz çünkü ispat eder.
    Boğaziçi Üniversitesi - Matematik

Çevrimdışı AtakanCİCEK

  • G.O Demirbaş Üye
  • ******
  • İleti: 264
  • Karma: +4/-0
  • Manisa
Ynt: Diyafont Denklemler Çalışma Soruları (70 Tane)
« Yanıtla #16 : Temmuz 18, 2019, 02:43:13 ös »
$22)$

$x>1$ sayıları için $x^6+2x^3+1<x^6+3x^3+1<x^6+4x^3+4$ olduğundan dolayı

 $(x^3+1)^2<x^6+3x^3+1<(x^3+2)^2$ olduğu için tam sayı çözümü yoktur.

$x^3<-3$ sayıları için $x^6+4x^3+4<x^6+3x^3+1<x^6+2x^3+1$  olduğundan dolayı

$(x^3+2)^2<x^3+6x+1<(x^3+1)^2$ olduğundan dolayı çözüm yoktur. Dolayısıyla $x$ $-1,0,1$ değerlerini alabilir.

$1)$ $x=-1$ ise $y^4=-1$ $y$ tam sayı olamaz

$2)$ $x=0$ ise $y^4=1$ yani $y=\pm1$ olur.

$3)$ $x=1$ ise $y^4=5$ olursa $y$ tam sayı olamaz. Dolayısıyla çözümler $(0,1)$ ve $(0,-1)$ olur.
Bir matematikçi sanmaz fakat bilir, inandırmaya çalışmaz çünkü ispat eder.
    Boğaziçi Üniversitesi - Matematik

Çevrimdışı AtakanCİCEK

  • G.O Demirbaş Üye
  • ******
  • İleti: 264
  • Karma: +4/-0
  • Manisa
Ynt: Diyafont Denklemler Çalışma Soruları (70 Tane)
« Yanıtla #17 : Temmuz 18, 2019, 02:44:10 ös »
$23)$

denklemde çıkarma işleminden  kurtulmak için bir dönüşüm yapalım. $a-1=x$ $b-1=y$ $c-1=z$ $xyz\mid (x+1).(y+1).(z+1)-1$ $x<y<z$ ve $x,y,z\in Z^+$ olur.

$xyz\mid (xy+x+y+1).(z+1)-1$

$xyz\mid xyz+xy+xz+yz+x+y+z+1-1$

$xyz\mid xy+xz+yz++x+y+z$ elde edilir. Buradan

$\frac{xy+xz+yz+x+y+z}{xyz}\in Z^+$ elde edilir. İfadeyi düzenlersek $\frac{1}{x}+\frac{1}{y}+\frac{1}{z}+\frac{1}{xy}+\frac{1}{xz}+\frac{1}{yz}$  olur. $x<y<z$ olduğundan

 $\frac{1}{x}>\frac{1}{y}>\frac{1}{z}>\frac{1}{xy}>\frac{1}{xz}>\frac{1}{yz}$ olur. Bu da bize $x$ yerine değer seçerek  İfadenin maksimum değerini bulma şansı verir. yani seçebileceğimiz en küçük üçlüyü seçerek işe koyulalım.

$(x,y,z)=(1,2,3)$ ise $1+\frac{1}{2}+\frac{1}{3}+\frac{1}{6}+\frac{1}{2}+\frac{1}{3}$ olur. Bu ise $2+\frac{5}{6}$ yani  ifadenin $3$ ten kesinlikle küçük olacağını söyler.

$(x,y,z)=(2,3,4)$ ise $\frac{1}{2}+\frac{1}{3}+\frac{1}{4}+\frac{1}{6}+\frac{1}{8}+\frac{1}{12}=\frac{35}{24}<2$ olur. Yani $x=2$ için ifadenin değeri $1$ dir.

$x\ge3$ için ise bu ifade en yüksek değerini $(3,4,5)$ için alır.

$\frac{1}{3}+\frac{1}{4}+\frac{1}{5}+\frac{1}{12}+\frac{1}{20}+\frac{1}{15}=\frac{59}{60}<1$   olur. bu da $x\ge2$  için çözüm olmadığını söyler.  Dolayısıyla $x=1$ için ifade $1$ veya $2$  olabildiğinden ve $x=2$ için ifadenin değeri $1$ olduğundan $3$ farklı durumda incelemek yeterlidir.

$1)$ $x=1$ için ifadenin değeri $1$ ise

$2y+2z+1+yz=yz$ 

$2y+2z+1=0$ olur. ki $2y+2z>0$ olduğundan doalyı mümkün değildir.

$2)$ $x=1$ için ifadenin değeri $2$ ise

$2y+2z+1=yz$ 

$2z+1=y.(z-2)$

$y=\frac{2z+1}{z-2}$

$\frac{2z+1+4-2z}{z-2}\in Z$ yani $z-2\mid 5$ olur.  $z>2$ olduğundan dolayı  $z\in \{ 3,7 \}$ olur.

$z=3$ ise $y=7$ olur. $z>y$ koşulu sağlanmadığından dolayı çözümü yoktur.

$z=7$ ise $y=3$ olur.  bu bir çözümdür. $(x,y,z)=(1,3,7)$ olduğundan $(a,b,c)=(2,4,8)$

$3)$ $x=2$ için ifadenin değeri 1 ise

$3z+3y+2+yz=2yz$

$3y+3z+2=yz$

$3z+2=y.(z-3)$

$\frac{3z+2}{z-3}\in Z$

$\frac{3z+2+9-3z}{z-3}\in Z$

 $z>2$ olduğundan ve $z\not\equiv 3$ olduğu için $z-3>0$ olmalıdır.

$\frac{11}{z-3}\in Z^+$ ve $Z\in \{4,14\}$ olmalıdır.

$z=4$ ise $y=14$ olur bu $z>y$ ile çelişir.

$z=14$ ise $y=4$ olur bu durumda $(x,y,z)=(2,4,14)$ bir çözümdür ve $(a,b,c)=(3,5,15)$ olarak bulunur.

bu bölme işlemini sağlayan üçlüler $(2,4,8)$ ve $(3,5,15)$  olmak üzere $2$ tanedir.
Bir matematikçi sanmaz fakat bilir, inandırmaya çalışmaz çünkü ispat eder.
    Boğaziçi Üniversitesi - Matematik

Çevrimdışı AtakanCİCEK

  • G.O Demirbaş Üye
  • ******
  • İleti: 264
  • Karma: +4/-0
  • Manisa
Ynt: Diyafont Denklemler Çalışma Soruları (70 Tane)
« Yanıtla #18 : Temmuz 18, 2019, 02:45:12 ös »
$24)$

$(x+1)^4-(x-1)^4=y^3$

$8x^3+8x=y^3$  yani $x^3+x$ tam küp olmalıdır. 

$x\ge 1$ için $x^3<x^3+x<x^3+3x^2+3x+1$ yani $x^3<x^3+x<(x+1)^3$ olduğundan çözüm yoktur.

$x\le -1$ için $x^3+3x^2+3x+1<x^3+x<x^3$ yani $(x+1)^3<x^3+x<x^3$ olduğundan dolayı  çözüm yoktur.

$x=0$ için $y=0$ çözümü bulunur. Denklemin tek çözümü $(0,0)$ olur.
Bir matematikçi sanmaz fakat bilir, inandırmaya çalışmaz çünkü ispat eder.
    Boğaziçi Üniversitesi - Matematik

Çevrimdışı AtakanCİCEK

  • G.O Demirbaş Üye
  • ******
  • İleti: 264
  • Karma: +4/-0
  • Manisa
Ynt: Diyafont Denklemler Çalışma Soruları (70 Tane)
« Yanıtla #19 : Temmuz 18, 2019, 02:45:53 ös »
$25)$

Varsayalım ki $p,q$ değerlerinin her ikisi de $3$ ten farklı olsun.

$3\mid p+q$ ancak ve ancak  $3\mid p^3-q^5$ olmalıdır.

$p,q\equiv\pm1(mod3)$ olur.

$1)$ $p\equiv q(mod3)$ ise $3\mid p^3-q^5$ olur ancak $3\mid p+q$ olmaz.

$2)$ $p\not\equiv q(mod3)$ ise $3\mid p+q$ olur ancak $3\mid p^3-q^5$ olmaz.

O halde $p$ veya $q$ $3$ olmalıdır. $p=3$ ise $27-q^5>0$ olduğundan $q=1$ olmalıdır. Fakat denklemi sağlamaz.

$q=3$ ise $p^3-243=(p+3)^2$ olur. $p>7$ için $p^3-243>(p+3)^2$ olur. $p<7$ için ise $p^3-243<(p+3)^2$ olur o halde yalnızca $p=7$ olabilir.

Denklemin çözümü $(7,3)$ olarak bulunur .
Bir matematikçi sanmaz fakat bilir, inandırmaya çalışmaz çünkü ispat eder.
    Boğaziçi Üniversitesi - Matematik

Çevrimdışı AtakanCİCEK

  • G.O Demirbaş Üye
  • ******
  • İleti: 264
  • Karma: +4/-0
  • Manisa
Ynt: Diyafont Denklemler Çalışma Soruları (70 Tane)
« Yanıtla #20 : Temmuz 18, 2019, 02:46:39 ös »
$26)$

$x^2-y!=2001$ denklemini  $x^2-2001=y!$ şeklinde yazalım. ardından denklemi $(mod9)$ altında inceleyelim.

$-2001\equiv 6(mod9)$ olduğunu not alalım.

$x^2\equiv \{0,1,4,7\} (mod9)$  olmalıdır.

$x^2+6\equiv y!\equiv  \{6,7,10,13\}(mod9)$ olduğu için $y!\not \equiv 0(mod9)$ olmalıdır. O halde $y<6$ olmalıdır.

$y\in \{1,2,3,4,5\}$ olur.

öncelikle $x^2\not \equiv \{2,3,7,8\}(mod10)$ ifadesine $(1)$ diyelim.

$y=1$ ise $x^2=2002$ olur $x^2\equiv 2(mod10)$ $(1)$ olamayacağından çözüm gelmez.

$y=2$ ise $x^2=2003$ olur $x^2\equiv 3(mod10)$ $(1)$ olamayacağından çözüm gelmez.

$y=3$ ise $x^2=2007$ olur $x^2\equiv 7(mod10)$ $(1)$ olamayacağından çözüm gelmez.

$y=4$ ise $x^2=2025$ olur $x=45$ olarak bulunur.

$y=5$ ise $x^2=2121$ olursa  $x^2=21.101$ olur. tam kare olamayacağından çözüm gelmez.

O halde denklemin tek çözümü $(45,4)$ olarak bulunur.
Bir matematikçi sanmaz fakat bilir, inandırmaya çalışmaz çünkü ispat eder.
    Boğaziçi Üniversitesi - Matematik

Çevrimdışı AtakanCİCEK

  • G.O Demirbaş Üye
  • ******
  • İleti: 264
  • Karma: +4/-0
  • Manisa
Ynt: Diyafont Denklemler Çalışma Soruları (70 Tane)
« Yanıtla #21 : Temmuz 18, 2019, 02:47:32 ös »
$28)$

Denklemde genelliği bozmadan $x\ge y$ alalım. Ve $x+y=k$ , $k\in Z$ dönüşümü yapalım.

$x^3-4x.(k-x)+(k-x)^3=-1$

$x^3-4xk+4x^2+k^3-3k^2x+3kx^2-x^3=-1$

$(3k+4).x^2+(-3k^2-4).x+k^3+1=0$

$\bigtriangleup=(-3k^2-4)^2-4.(3k+4).(k^3+1)$

$9k^4+24k^2+16-12k^4-12k-16k^3-16$

$\bigtriangleup=-3k^4-16k^3+24k^2-12k$ olur.

$k\ge3$ için $k^2>8$ yani $3k^4>24k^2$ olduğundan dolayı $\bigtriangleup<0$ olur.

$k<3$ olmalıdır.

$k\le -7$ için ise $\bigtriangleup \le-65<0$ olur.

$k\in \{-6,-5,-4,-3,-2,-1,0,1,2\}$ olmalıdır.

$k=2$ için $\bigtriangleup =-48-128+96-24<0$ olduğundan çözüm yoktur.

$k=1$ için $\bigtriangleup =-7<0$

$k=0$ için $\bigtriangleup =0$ olur denenmelidir.

$k=-1$ için $\bigtriangleup=49$ olur tam kare olduğundan sağlayabilir.

$k=-2$ için $\bigtriangleup=200$ olur tam kare olmadığından çözüm yoktur.

$k=-3$ için $\bigtriangleup =441$ olur tam kare olduğundan sağlayabilir.

$k=-4$ için $\bigtriangleup =688$ olur tam kare olmadığından çözüm yoktur.

$k=-5$ için $\bigtriangleup =785$ olur tam kare olmadığından çözüm yoktur.

$k=-6$ için $\bigtriangleup =504$ olur tam kare olmadığından çözüm yoktur.

O halde $k\in \{0,-1,-3\}$ şeklinde kümemizi elde ederiz.

$1)$ $k=0$ ise $4x^2-4x+1=0$ $x\not\in Z$ olduğundan çözümü yoktur.

$2)$ $k=-1$ ise $x^2-7x=0$ $x\in \{0,7\}$ olabilir.

$3)$ $k=-3$ ise $-5x^2-31x-26=0$ $5x^2+31x+26=0$ olmaldıır. $x=-1$ olabilir.

Olası çözümler $x>y$ için $(0,-1),(7,-8),(-1,-2)$ olur. Verilen denklemlde denersek  sadece $(0,-1)$ ilk denklemi sağlar

Denklemin çözüm kümesi $\{ (0,-1),(-1,0)\}$ olarak bulunur.
Bir matematikçi sanmaz fakat bilir, inandırmaya çalışmaz çünkü ispat eder.
    Boğaziçi Üniversitesi - Matematik

Çevrimdışı AtakanCİCEK

  • G.O Demirbaş Üye
  • ******
  • İleti: 264
  • Karma: +4/-0
  • Manisa
Ynt: Diyafont Denklemler Çalışma Soruları (70 Tane)
« Yanıtla #22 : Temmuz 18, 2019, 02:48:23 ös »
$32)$

$p>1$ olduğundan dolayı $p^3+p^2+11p+2>15$ olmalıdır. 

tam sayılar 6 ya bölündüklerinde $\{6k,6k+1,6k+2,6k+3,6k+4,6k+5\}$ olabilir.

$p=6k$ olursa $p$ asal olamaz.

$p=6k+1$  olursa sonsuz tane $p$ vardır.

$p=6k+2$ olursa $p$ asal ise $p=2$ olmalıdır.

$p=6k+3$ olursa $p$ asal ise $p=3$ olmalıdır.

$p=6k+4$ olursa $p$ asal olamaz.

$p=6k+5$ olursa sonsuz tane $p$ vardır.

$1)$   $p=6k+1$ için $q=p^3+p^2+11p+2\equiv 3(mod6)$ ve $q>3$ olduğundan çözümü yoktur.

$2)$   $p=6k+5$ için $q=p^3+p^2+11p+2\equiv 3(mod6)$ ve $q>3$ olduğundan çözümü yoktur.

$3)$  $p=2$ için $36=q$ olur $q$ asal değildir.

$4)$ $p=3$ için $71=1$ olur $q$ asaldır.

Tek çözümü $(3,71)$ olarak bulunur.
Bir matematikçi sanmaz fakat bilir, inandırmaya çalışmaz çünkü ispat eder.
    Boğaziçi Üniversitesi - Matematik

Çevrimdışı AtakanCİCEK

  • G.O Demirbaş Üye
  • ******
  • İleti: 264
  • Karma: +4/-0
  • Manisa
Ynt: Diyafont Denklemler Çalışma Soruları (70 Tane)
« Yanıtla #23 : Temmuz 18, 2019, 02:48:59 ös »
$33)$

$x^3-y^3$ ifadesi görüldüğünde yapılan klasik $x=y+m$ dönüşümünü yapalım. $x^3-y^3=2y^2+1>0$ olduğundan $x^3>y^3$ yani $x>y$ olur.  Bu nedenle $m\in Z^+$ olmalıdır.

$(y+m)^3-y^3=2y^2+1$

$3my^2+3m^2y+m^3=2y^2+1$

$(3m-2).y^2+3m^2y+m^3-1=0$

$\bigtriangleup=9m^4-4.(m^3-1).(3m-2)=-3m^4+8m^3+12m-8$

$m\ge 4$ için $\bigtriangleup\le-84<0$  olur. O halde $m<3$ olmalıdır. $m\in \{1,2,3\}$

$m=1$ ise $y^2+3y=0$ olur ve $y=0$ ve $y=-3$ çözümleri oluşur. $(1,0)$ ve $(-3,-2)$

$m=2$ ise $4y^2+12y+7=0$ olur tam sayılarda çözümü yoktur.

$m=3$ ise $7y^2+27y+26=(y+2).(7y+13)=0$ olur. $y=-2$ çözümü çıkar. $(1,-2)$

Dolayısıyla denklemin çözümleri $\{(1,0),(1,-2),(-3,-2)\}$ olur.
Bir matematikçi sanmaz fakat bilir, inandırmaya çalışmaz çünkü ispat eder.
    Boğaziçi Üniversitesi - Matematik

Çevrimdışı AtakanCİCEK

  • G.O Demirbaş Üye
  • ******
  • İleti: 264
  • Karma: +4/-0
  • Manisa
Ynt: Diyafont Denklemler Çalışma Soruları (70 Tane)
« Yanıtla #24 : Temmuz 18, 2019, 02:49:44 ös »
$34)$

$x=1$ için $p=5$ açık çözümdür.

Soruya baktığımızda amacımızın  $x^4+4^x=p$ ifadesini çarpanlarına ayırabilmek olabileceği akla gelmelidir.  $x>1$ için çarpanlara ayıralım.

varsayalım ki $x=2k$ ,$k\in Z^+$  olsun.

$16k^4+4^2k=p$ olur ve buradan $p$ $4$ ile bölüneceğinden dolayı $x=2k$ olamaz.

$x$ in tek olduğunu bulduk şimdi ise $4^x\equiv 4(mod5)$ ancak ve ancak $x=2k+1$  olduğuna dikkat çekelim.

$x^4\equiv\{0,1\} (mod4)$ olduğu açıktır.

$x^4+4^x\equiv 0(mod5)$ olmaması için $x^4\equiv 0(mod5)$ olur. yani $x\equiv 0(mod5)$ olmalıdır.  Aynı anda sayı tek olduğundan dolayı $x=10k+5$ olarak bulunur.

$x^4+4.4^{10k+4}$ şeklinde yazıp  $2^{5k+2}=y$ dönüşümü yaparsak

 $x^4+4y^4=x^4+4x^2y^2+4y^4-4x^2y^2=(x^2-2xy+2y^2).(x^2+2xy+2y^2)$  olur.

Dolayısıyla denklemin tek çözümü $(1,5)$ olur.
Bir matematikçi sanmaz fakat bilir, inandırmaya çalışmaz çünkü ispat eder.
    Boğaziçi Üniversitesi - Matematik

Çevrimdışı AtakanCİCEK

  • G.O Demirbaş Üye
  • ******
  • İleti: 264
  • Karma: +4/-0
  • Manisa
Ynt: Diyafont Denklemler Çalışma Soruları (70 Tane)
« Yanıtla #25 : Temmuz 18, 2019, 02:51:07 ös »
$35)$

İlk Yol

denklemde bilinmeyenlerin dereceleri çift olduğundan dolayı genelliği kaybetmeden $a\ge0$ ,$b\ge0$ ve $c\ge 0$ alınabilir.

$c^2+1=(a^2-1).(b^2-1)$ denkleminde $a=b=c=0$ açık çözümü olduğu görülebilmektedir.

$a,b,c>0$ olduğunu kabul edelim.

Şimdi $c^2+1$ için geçerli olan özellikler bulalım.

$c=2k$ olursa $c^2+1\equiv 1(mod4)$ olur.

$y=2c+1$ olursa $c^2+1 \equiv 2(mod4)$ olur. $(1)$

Şimdi $c$ nin $2$ dışında asal çarpanlarının tamamının $4k+1$ formunda olduğunu gösterelim.$(2)$

$p\mid c^2+1$ olsun. $c^2\equiv -1(modp)$ olur

$c^4\equiv1(modp)$

 $p\mid c^2+1$ olduğundan $(c,p)=1$ dir.

Fermat teoreminden $c^{p-1}\equiv1(modp)$

Buradan $4$'ün $p-1$ in en küçük katı olduğunu görmek mümkündür $4\mid p-1$ elde edilir.

yani $p=4k+3$ formunda çarpan ya da sayı $4$ ün katı olamaz.

$(a^2-1).(b^2-1)$ ifadesine bakarsak $a$ ya da $b$ çift olursa $c^2+1$ ifadesinin $4k+3$ formunda çarpanı olacağı için mümkün değildir.

$a$ ya da $b$ den biri tek olursa da bu sefer sayı $4$ ün katı olacağı için mümkün değildir. O halde tek çözüm $(0,0,0)$ olarak bulunur.

İkinci Yol

denklemde bilinmeyenlerin dereceleri çift olduğundan dolayı genelliği kaybetmeden $a\ge0$ ,$b\ge0$ ve $c\ge 0$ alınabilir.

$c^2+1=(a^2-1).(b^2-1)$ $(0,0,0)$ çözümü açıktır.

$c^2+1$ ifadesi $4$ ün katı olamayacağından dolayı $a$ ve $b$ tek sayılar olamaz.

$a=2a_1$ ve $b=2b_1$ olacak şekilde $a_1\in Z^+$ ve $b_1 \in Z^+$ olmalıdır.

denklemi yazıp tekrar düzenleyelim.

$c_1^2=4a_1^2b_1^2-a_1^2-b_1^2$

Bu denklemi de $(mod4)$ altında inceleyecek olursak eğer $a_1$ tek sayı ise $a_1^2\equiv 1(mod4)$ olur.  $a_1$ veya $b_1$ den herhangi biri veya ikisi tek olursa $c^2\equiv \{2,3\}(mod4)$ olacağından dolayı mümkün değildir.

o halde $a_1=2a_2$ $b_1=2b_2$ ve $c_1=2c_2$ olur. Bunu da düzenleyecek olursak $c_2^2=16a_2^2b_2^2-a_2^2-b_2^2$ olur. ve buradan sonsuz dizi elde edilir.

$c>c_1>c_2>c_3>...>0$ şeklinde bir sonsuz dizi elde edilir. Fakat  $c$ bir tam sayı olduğu için $(0,c)$ aralığında sonlu sayıda tam sayı olacağından sonsuz tam sayı dizisi mümkün değildir.

O halde denklemin tek çözümü $(0,0,0)$ olarak bulunur.
Bir matematikçi sanmaz fakat bilir, inandırmaya çalışmaz çünkü ispat eder.
    Boğaziçi Üniversitesi - Matematik

Çevrimdışı AtakanCİCEK

  • G.O Demirbaş Üye
  • ******
  • İleti: 264
  • Karma: +4/-0
  • Manisa
Ynt: Diyafont Denklemler Çalışma Soruları (70 Tane)
« Yanıtla #26 : Temmuz 18, 2019, 02:51:32 ös »
$36)$

$9^x-3^x$ ifadesini $3^x=a$ ,$a\in Z^+$ şeklinde düzenleyelim.

$a^2-a=y^4+2y^3+y^2+2y$ , $4a^2-4a+1=4y^4+8y^3+4y^2+8y+1$ şeklinde yazılabilir.

$(2a-1)^2=4y^4+8y^3+4y^2+8y+1$

$(2y^2+my+1)^2=4y^4+4my^3+(4+m^2).y^2+2my+1$ olur. yukarıdaki polinom ifade ile $y^3$ lü terimlerin katsayılarını eşitlersek $m=2$ almamız gerektiğini görürüz.

$(2y^2+2y)^2=4y^4+8y^3+4y^2$

$(2y^2+2y+1)^2=4y^4+8y^3+8y^2+4y+1$  ifadeleri $y>1$ için açıkça  $(2y^2+2y)^2<4y^4+8y^3+4y^2+8y+1<(2y^2+2y+1)^2$ olduğundan  dolayı tam kare olması mümkün değildir.

$y=0$ veya $y=1$ olmalıdır.

$1)$ $y=0$ ise $9^x-3^x=0$ $3^x.(3^x-1)=0$ yani $x=0$ çözümü elde edilir. $(0,0)$ olur.

$2)$  $y=1$ ise $a^2-a=6$ $a^2-a-6=$ $a>0$ olduğundan $a=3$,$3^x=3$, $x=1$ çözümü elde eidlir. $(1,1)$ olur.

O halde denklemin çözüm kümesi $\{(0,0),(1,1)\}$ olarak bulunur.
Bir matematikçi sanmaz fakat bilir, inandırmaya çalışmaz çünkü ispat eder.
    Boğaziçi Üniversitesi - Matematik

Çevrimdışı AtakanCİCEK

  • G.O Demirbaş Üye
  • ******
  • İleti: 264
  • Karma: +4/-0
  • Manisa
Ynt: Diyafont Denklemler Çalışma Soruları (70 Tane)
« Yanıtla #27 : Temmuz 18, 2019, 02:52:11 ös »
$37)$

$(x-y).(x+y)=2^k$ olduğundan dolayı $x+y=2^m$ , $x-y=2^n$ olur.

$x=2^{m-1}+2^{n-1}$ ve $y=2^{m-1}-2^{n-1}$ olarak bulunur.

burada $y$ tek bir pozitif tam sayı olduğunu kullanırsak  $m=1$ ya da $n=1$ olmalıdır.

$m=1$ olursa $y=1-2^{n-1}\le0$ ki pozitif olmasıyla çelişir. O halde $n=1$ olmalıdır.

$x-y=2$ elde edileceğinden dolayı $x$ ve $y$ aynı anda ortak  asal çarpan içeremez

$x=3^k$, $y=3^l$ veya $x=5^k$ ,$y=3^l$ olmalıdır.

$x=3^k$ ,$y=5^l$ için $5^l=2^{m-1}-1$ olacağını görürüz. $(mod4)$ altında bakarsak

$2^{m-1}\equiv 2(mod4)$ olur. $m=2$ tek çözümüdür. $m=2$ ve $n=1$ yerine koyulduğunda $(x,y)=(3,1)$ ortaya çıkar.

$x=5^k$ ve $y=5^l$ ise $3^m\equiv\{1,3\}(mod8)$ olmalıdır.

$2^{m-1}=\equiv{2,4}(mod8)$ olduğundan dolayı $m=2$ ya da $m=3$ olabilir.

$m=2$ için $x=3$ olur fakat $x=5^k$ formunda kabul etmiştik.

$m=3$ için $x=5$ ve $y=3$ olur.  O halde $(5,3)$ te bir çözümdür.

Denklemin çözüm kümesi $\{(3,1),(5,3)\}$ olarak bulunur.
Bir matematikçi sanmaz fakat bilir, inandırmaya çalışmaz çünkü ispat eder.
    Boğaziçi Üniversitesi - Matematik

Çevrimdışı AtakanCİCEK

  • G.O Demirbaş Üye
  • ******
  • İleti: 264
  • Karma: +4/-0
  • Manisa
Ynt: Diyafont Denklemler Çalışma Soruları (70 Tane)
« Yanıtla #28 : Temmuz 18, 2019, 02:53:10 ös »
$47)$

$x^3\equiv -1,0,1 (mod7)$ olduğunu kullanalım. $18+c^3-b^3\in 2,3,4,5,6(mod7)$ elde edilebilir. yani $a!\not\equiv 0(mod7)$ olduğundan dolayı $a<7$ olduğunu buluruz.

$1)$ $a=2$ için $b^3-c^3=16$ olur.  $c\ge 2$ için $b>c$ olduğundan dolayı en az $b=c+1$ olabilir. bunun için ise $3c^2+3c+1$ $c\ge2$ için çözümsüzdür. $c=1$ olması durumunda ise $7=16$ olacağından dolayı çözümü yoktur. $c+2$ olsa idi $6c^2+12c+8>16$ olacağından dolayı $a=2$ için çözümü yoktur.

$2)$ $a>2$ için $3\mid b^3-c^3$ olduğundan dolayı $9\mid b^3-c^3$ te olması gerektiğinden dolayı $a\ge6$ olması gerektiği bulunur. Dolayısıyla $a=6$  olmalıdır.

$a=6$ için $c^3-b^3=702=2.3^3.13$ olur. $(c-b).(c^2+bc+b^2)=702$ yani $(c-b).[(c-b)^2+3bc]$ $3\mid c-b$ olur. dolayısıyla $3x=c-b$ , $x\in Z^+$ vardır.  $3x.(9x^2+3bc)=2.3^3.13$ olur. ve $x.(3x^2+bc)=2.3.13$ olarak düzenlenir. $x>3$ için $3x^2+bc>27$ olur ve
$x.(3x^2+bc)>81$ yani $x.(3x^2+bc)>78$ olduğundan çözüm yoktur. $x=1$ veya $x=2$ olmalıdır.

$x=1$ için $1.(3+bc)=78$ $bc=75$ $(c-b)=3.1$ $b.(b+3)=78$ olur ancak çözüm gelmez.

$x=2$ için $2.(12+bc)=78$ $12+bc=39$ $bc=27$ olur. $c-b=3.2$ $bc=27$ $b.(b+6)=27$  $b^2+6b-27=0$ yani $b=3$ $c=9$ çözümü gelir.

O halde denklemin çözümü $(6,3,9)$ olmalıdır.   
Bir matematikçi sanmaz fakat bilir, inandırmaya çalışmaz çünkü ispat eder.
    Boğaziçi Üniversitesi - Matematik

Çevrimdışı AtakanCİCEK

  • G.O Demirbaş Üye
  • ******
  • İleti: 264
  • Karma: +4/-0
  • Manisa
Ynt: Diyafont Denklemler Çalışma Soruları (70 Tane)
« Yanıtla #29 : Temmuz 18, 2019, 02:53:40 ös »
$48)$

$x^3\equiv -1,0,1(mod3)$ olduğunu ve denklemden dolayı $k>0$ için  $m^3-n^3\equiv 6(mod9)$ olmalıdır. ancak $m^3-n^3\in -2,-1,0,1,2(mod9)$ olduğundan dolayı denklem sağlanamaz. $k=0$ olmalıdır.

$m^3-n^3=124$ denklemini çözmemiz yeterlidir.

$(m-n).[(m-n)^2+3mn]=124$ 

\begin{equation*}
\begin{cases}
m-n=1,
\\
(m-n)^2+3mn=124
\end{cases}
\end{equation*}

\begin{equation*}
\begin{cases}
m-n=2
\\
(m-n)^2+3mn=62
\end{cases}
\end{equation*}

\begin{equation*}
\begin{cases}
m-n=4,
\\
(m-n)^2+3mn=31
\end{cases}
\end{equation*}

Bu sistemlerden ilkini çözmek istersek $3mn=123$ yani $mn=39$ olur $39$ iki ardışık sayının çarpımı olarak yazılamayacağından dolayı  ilk sistemin çözümü yoktur.

$2.$ sistemde $3mn=58$ olduğundan dolayı $mn\notin Z$ olur ikinci sistemin de çözümü yoktur.

$3.$ sistemde ise $3mn=45$ $mn=15$ olur. $n.(n+4)=5$ $n^2+4n-5=0$ $n=1$ çözümü olur. $m=5$ te olması gerektiğinden dolayı $(5,1,0)$  denklemin tek çözümü olur.
Bir matematikçi sanmaz fakat bilir, inandırmaya çalışmaz çünkü ispat eder.
    Boğaziçi Üniversitesi - Matematik

 


Sitemap 1 2 3 4 5 6 7 8 9 10 11 12 13 14 15 16 17 18 19 20 21 22 23 24 25 26 27 28 29 30 31 32 33 34 35 36 37 
SimplePortal 2.3.3 © 2008-2010, SimplePortal